Difference between revisions of "G285 2021 MC10B"

m
m
Line 20: Line 20:
  
 
[[G285 MC10B Problems/Problem 3|Solution]]
 
[[G285 MC10B Problems/Problem 3|Solution]]
 +
 +
==Problem 4==
 +
 +
==Problem 5==

Revision as of 16:56, 12 May 2021

Problem 1

Find $\left \lceil {\frac{3!+4!+5!+6!}{2+3+4+5+6}} \right \rceil$

$\textbf{(A)}\ 42\qquad\textbf{(B)}\ 43\qquad\textbf{(C)}\ 44\qquad\textbf{(D)}\ 45\qquad\textbf{(E)}\ 46$

Solution

Problem 2

If $deg(Q(x))=3$, and $deg(K(x))=2$, and $Q(x)=(x-2)K(x)$, what is $deg(Q(x)-2K(x))$?

$\textbf{(A)}\ 0\qquad\textbf{(B)}\ 1\qquad\textbf{(C)}\ 2\qquad\textbf{(D)}\ 3\qquad\textbf{(E)}\ 4$

Solution

Problem 3

A convex hexagon of length $s$ is inscribed in a circle of radius $r$, where $r \neq s$. If $\frac{s}{2r}=\frac{21}{29}$, and $rs=58$, find the area of the hexagon.

$\textbf{(A)}\ 42\qquad\textbf{(B)}\ 60\qquad\textbf{(C)}\ 84\qquad\textbf{(D)}\ 90\qquad\textbf{(E)}\ 120$

Solution

Problem 4

Problem 5